Raj tells his daughter, “Seven years ago, I was seven times as old as you were then. Also, three years from now, I shall be three times as old as you will be.” Find the present ages of Raj and his daughter. Also, verify the present age of Raj and his daughter graphically

Answers

Answer 1

The present ages of Raj and his daughter are respectively: 42 years and 12 years.

How to solve Algebra Word Problems?

Present Age of Raj =y years

Present Age of daughter =x years

According to Question :

7 Years ago,

y − 7 = 7(x − 7)

⇒ 7x − y − 42 = 0.............(1)

And 3 Years from now

y + 3 = 3(x + 3)

⇒ 3x − y + 6 = 0............(2)

From eq (1) and eq (2)

Subtract eq 2 from eq 1 to get:

7x − 3x − y + y − 42 − 6 = 0

⇒ 4x = 48

⇒ x = 12

Putting x = 12 in Equation (2). we get,

(3 × 12) − y + 6 = 0

⇒ y = 42

Read more about Algebra Word Problems at: https://brainly.com/question/21405634

#SPJ1

Raj Tells His Daughter, Seven Years Ago, I Was Seven Times As Old As You Were Then. Also, Three Years

Related Questions

Joshua's mail truck travels 14 miles every day he works


and is not used at all on days he does not work. At the


end of his 100th day of work the mail truck shows a


mileage of 76,762. Model Joshua's truck mileage as a


function of the number of days he has worked. When


will he reach 100,000 miles?

Answers

Solving the equation, Joshua will reach 100,000 miles after approximately 1,760 days of work.

To model Joshua's truck mileage as a function of the number of days he has worked, we can use the following equation:

Mileage (M) = 14 * Number of days worked (D) + Initial Mileage (I)

First, we need to determine the initial mileage on the mail truck. To do this, we can use the information given for his 100th day of work:

76,762 = 14 * 100 + Initial Mileage
76,762 = 1,400 + Initial Mileage
Initial Mileage (I) = 76,762 - 1,400
Initial Mileage (I) = 75,362

Now we can rewrite the equation as:

Mileage (M) = 14 * Number of days worked (D) + 75,362

To find when Joshua will reach 100,000 miles, we can set M equal to 100,000 and solve for D:

100,000 = 14 * D + 75,362
24,638 = 14 * D
D ≈ 24,638 / 14
D ≈ 1,759.857

Since Joshua cannot work a fraction of a day, he will reach 100,000 miles after approximately 1,760 days of work.

Know more about equation here:

https://brainly.com/question/29174899

#SPJ11

he figure below is a net for a rectangular prism. Side a = 62 centimeters, side b = 21 centimeters, and side c = 16 centimeters. What is the surface area of this figure?

Answers

The surface area of the rectangular prism is 4960 cm².

The rectangular prism can be divided into six rectangular faces, with opposite faces having the same area. To find the surface area, we need to calculate the area of each face and add them up.

The net shows three rectangles with dimensions of 62 cm x 21 cm, 62 cm x 16 cm, and 21 cm x 16 cm.

Therefore, the surface area of the rectangular prism is:

Area of the first rectangle = 62 cm x 21 cm = 1302 cm²

Area of the second rectangle = 62 cm x 16 cm = 992 cm²

Area of the third rectangle = 21 cm x 16 cm = 336 cm²

Total surface area = 2(Area of first rectangle) + 2(Area of second rectangle) + 2(Area of third rectangle)

= 2(1302) + 2(992) + 2(336)

= 2604 + 1984 + 672

= 4960 cm²

To learn more about surface follow the link:

https://brainly.com/question/29298005

#SPJ1

Question
Write the product using exponents.

(−13)⋅(−13)⋅(−13)

Answers

Answer:

(-13)^3

Step-by-step explanation:

Exponents can be used for repeated multiplication.

In this case, the number "negative 13" is repeated several times, all connected with multiplication.

There are a total of three "negative 13"s being multiplied together ("negative 13" appears three times on the page).

To rewrite using exponents, we would write one of the following:

(-13)^3

[tex](-13)^3[/tex]

Determine whether the graph shows a positive correlation, a negative correlation, or no correlation. If there is a positive or negative correlation, describe its meaning in the situation. Domestic Traveler Spending in the U.S., 1987-1999 Spending (dollars in billions) A graph titled Domestic Traveler Spending in the U S from 1987 to 1999 has year on the x-axis, and spending (dollars in billions) on the y-axis, from 225 to 450 in increments of 25. Year Source: The World Almanac, 2003 a. positive correlation; as time passes, spending increases. b. no correlation c. positive correlation; as time passes, spending decreases. d. negative correlation; as time passes, spending decreases.

Answers

There is a positive correlation and as such as time passes, spending increases.

Checking the correlation of the graph

The descriptions of the graph from the question are given as

Year (x - axis): 1987 to 1999Spending (y - axis, dollars in billions) 225 to 450 in increments of 25.

From the above statements, we can make the following summary

As the year increase, the spending also increase

The above summary is about the correlation of the graph

And it means that there is a positive correlation and as such as time passes, spending increases.

Read more about correlation at

https://brainly.com/question/1564293

#SPJ1

find the next two terms in sequence?

Answers

Answer:

83 and 99 The sequence is going in +16 so the answer is 83 and 99 sorry if I am a bad explainer I’m new to this app :(

Step-by-step explanation:

Joey needs to buy chocolate milk they only sell it in pint containers how many pint containers of chocolate milk should he buy

Answers

To determin how many pint containers of chocolate milk Joey should buy, we need to know how much chocolate milk he wants in total.

Let's assume Joey wants to buy "x" pints of chocolate milk. Then, the total amount of chocolate milk he will get in quarts can be calculated as:

Total amount of chocolate milk = x pints/2 pints per quart = x/2 quarts

If Joey has a specific total amount of chocolate milk he wants to buy, we can solve for "x". For example, if Joey wants to buy 3 quarts of chocolate milk, we can set up the following equation:

x/2 = 3

Multiplying both sides by 2, we get:

x = 6

So, Joey needs to buy 6 pint containers of chocolate milk to get 3 quarts in total.

If Joey has a different desired amount of chocolate milk, we can adjust the equation accordingly to find the number of pint containers he needs to buy

To know more about Multiplying refer here

https://brainly.com/question/42663642#

#SPJ11

For the cost function C(x) = 6000 + 242 + 0.005.03 find: A) The production level that will minimize the average cost. B) The minimal average cost.

Answers

To find the production level that will minimize the average cost, we need to differentiate the cost function with respect to x and set it equal to zero. So:

C'(x) = 0.005x^2 + 242x + 6000
0 = 0.005x^2 + 242x + 6000
Using the quadratic formula, we get:
x = (-242 ± sqrt(242^2 - 4(0.005)(6000))) / (2(0.005))
x = (-242 ± sqrt(146416)) / 0.01
x = (-242 ± 382) / 0.01
x = -14,000 or 27,000

Since the production level cannot be negative, we can discard the negative solution and conclude that the production level that will minimize the average cost is 27,000 units.

To find the minimal average cost, we need to plug the production level back into the cost function and divide by the production level. So:

C(27,000) = 6000 + 242(27,000) + 0.005(27,000)^2
C(27,000) = 6,594,000
Average cost = C(27,000) / 27,000
Average cost = 6,594,000 / 27,000
Average cost ≈ 244.22

Therefore, the minimal average cost is approximately $244.22.
To answer your question, first, let's correct the cost function, which should be in the form of C(x) = Fixed cost + Variable cost. Assuming it is C(x) = 6000 + 242x + 0.005x^2.

A) To find the production level that will minimize the average cost, we need to first determine the average cost function, which is AC(x) = C(x)/x. So, AC(x) = (6000 + 242x + 0.005x^2)/x.

Now, find the first derivative of AC(x) concerning x, and set it equal to zero to find the minimum point:

d(AC(x))/dx = 0

The first derivative of AC(x) is:

d(AC(x))/dx = (242 + 0.010x - 6000/x^2)

Setting this to zero and solving for x will give us the production level that minimizes the average cost:

242 + 0.010x - 6000/x^2 = 0

Now, you can solve for x using numerical methods, such as Newton-Raphson or others. After solving for x, you will get the production level that minimizes the average cost.

B) To find the minimal average cost, plug the production level x you found in part A into the average cost function, AC(x):

Minimal Average Cost = AC(production level)

This will give you the minimal average cost for the given cost function.

To learn more about production visit;

brainly.com/question/30333196

#SPJ11

Helppppppppppppppppp

Answers

Answer:
- point in org. Figure: (3,5)
- point in final figure: (-5,5)

Explanation:
Translate just means move across, since it said moves to the left we make that point x go across the x axis.

The equation y=mx+b
is used to express the equation of a line. Which solution is a correct way to solve this equation for m
in terms of y
?

Answers

The  correct way to solve this equation for m in terms of y is  m = b - y/x

How to determine the value

It is important to note that subject of formula is the variable that is made to stand alone in an equation.

It is described as the variable that is being worked out in an equation.

The subject of formula in an equation is worked to stand alone on on end of the equality sign.

Example of equations

x = y - 2

The variable 'x' is the subject of formula

From the information given, we have that;

y= mx+b

collect the terms

mx = b - y

Divide by the coefficient

m = b - y/x

The equation for m is m = b - y/x

Learn about subject of formula at: https://brainly.com/question/10643782

#SPJ1

The complete question:

The equation y=mx+b is used to express the equation of a line. Which solution is a correct way to solve this equation for m in terms of y?

HELP ME PLEASE I WILL GIVR BRAINLIEST TO THE FASTED CORRECT ANSWER PLEASE HELP ME FAST AND TY

Answers

Base is 3, height is 12. 3 x 12 is 36, and 36/3 is 12


Answer: 12

At her job, Avery earns $120 per week plus a one-time $300 bonus. Janelle teaches art lessons and earns $24 per week plus a $60 art supply fee for each student she teaches. a. System of equations:

Answers

The system of equations to describe the earnings by Avery and Janelle would be:

Avery's earnings: y = 120x + 300

Janelle's earnings: y = 24x + 60s

How to find the system of equations ?

The problem provides two scenarios with different methods for earning money. Avery earns a fixed amount of $120 each week, in addition to a one-time bonus of $300. To represent this situation as an equation, we can use the formula:

y = 120x + 300

where y is Avery's total earnings, x is the number of weeks she works, and 300 is the one-time bonus she receives.

For Janelle, her earnings consist of a fixed weekly rate of $24 plus a variable amount based on the number of students she teaches.

We can represent Janelle's earnings as an equation using the formula:

y = 24x + 60s

where y is Janelle's total earnings, x is the number of weeks she works, and s is the number of students she teaches.

Find out more on system of equations at https://brainly.com/question/13729904

#SPJ1

The guidance department has reported that of the senior class 2. 3% are members of key club 8. 6% are enrolled in AP physics and 1. 9% are in both

Answers

The percentage is 9.0% of the senior class are either members of the Key Club, enrolled in AP Physics, or both.

We need to find the percentage of seniors who are either members of the Key Club, enrolled in AP Physics, or both. We can use the formula:

Total percentage = Key Club percentage + AP Physics percentage - Both percentage

Step 1: Identify the given percentages
Key Club percentage = 2.3%
AP Physics percentage = 8.6%
Both percentage = 1.9%

Step 2: Apply the formula
Total percentage = 2.3% + 8.6% - 1.9%

Step 3: Calculate the result
Total percentage = 9.0%

So, 9.0% of the senior class are either members of the Key Club, enrolled in AP Physics, or both.

Learn more about percentage,

https://brainly.com/question/24877689

#SPJ11

to find out whether a new serum will arrest leukemia, 9 mice, all with an advanced stage of the disease, are selected. five mice receive the treatment and 4 do not. survival times, in years, from the time the experiment commenced are as follows: treatment 2.1 5.3 1.4 4.6 0.9 no treatment 1.9 0.5 2.8 3.1 at the 0.05 level of significance, can the serum be said to be effective? assume the two populations to be normally distributed with equal variances.

Answers

The serum be said to be effective can't be concluded, since the test statistic is less than the critical value, we fail to reject the null hypothesis.

Let [tex]n_A[/tex] denotes the number of mice which receiving treatment. Therefore,

[tex]n_A[/tex] = 5,

Let [tex]n_B[/tex] denotes the number of mice which do not receive treatment. Therefore, [tex]n_B[/tex] = 4

Survival times for the mice receiving the treatment are: 2.1; 5.3; 1.4; 4.6; 0.9

Survival times for the mice not receiving the treatment are: 1.9; 0.5; 2.8; 3.1

Let [tex]x_A[/tex] be the mean of survival time for the mice receiving the treatment and [tex]x_B[/tex] be the mean of survival time for the mice not receiving the treatment.

We have: [tex]x_A[/tex] = 2.86

[tex]x_B[/tex] = 2.075

Standard deviation be:

[tex]S_A=\sqrt{\frac{\sum (x_a-x_A)^2}{n_A-1} }[/tex]

[tex]=\sqrt{\frac{[(2.1-2.86)^2+(5.3-2.86)^2+(1.4-2.86)^2+(4.6-2.86)^2+(0.9-2.86)^2]}{4} }[/tex]

= 1.971

[tex]S_B=\sqrt{\frac{\sum (x_b-x_B)^2}{n_B-1} }[/tex]

[tex]=\sqrt{\frac{[(1.9-2.08)^2+(0.5-2.08)^2+(2.8-2.08)^2+(3.1-2.08)^2]}{3} }[/tex]

= 1.167

[tex]\mu_A[/tex] and [tex]\mu_B[/tex]  are population means for the groups receiving the treatment and not receiving the treatment respectively.

Level of significance is α = 0.05

If P-value is less then 0.05, we will reject [tex]H_o[/tex]

The test statistic is,

[tex]t=\frac{(x_A-x_B)-(\mu_A-\mu_B)}{s_p\sqrt{\frac{1}{n1} +\frac{1}{n2} } }[/tex]

[tex]=\frac{2.86-2.07)-(0)}{1.674388\sqrt{\frac{1}{5} +\frac{1}4} } }[/tex]

= 0.79/1.123

t = 0.70

Degrees of freedom is,

[tex]d_f=n_A+n_B-2[/tex]

= 5 + 4 - 2

= 7.

According to the value in the table, the test's critical value is 1.895.

We are unable to reject the null hypothesis since the test statistic is less than the threshold value.

We thus cannot draw the conclusion that the serum is working.

Learn more about the Test statistics:

https://brainly.com/question/15980493

#SPJ4

Can someone help with this math equation from study island…????

Answers

The solution of the exponents is shown below.

What is the solution of the exponents?

Exponents are mathematical shorthand for multiplying a number by itself a certain number

We have that;

5^n = 1

5^n = 5^0

n = 0

2) 2^-7/2^n = 2^2

2^-7 - n = 2^2

-7 - n = 2

-n = 2 + 7

n = -9

3) 6^5 * 6^n = 6^1

6^ 5 + n = 6^1

5 + n = 1

n = 1 - 5

n = -4

4) (8^n)^7 = 8^21

8^7n = 8^21

7n = 21

n = 3

5) 4^n = (1/4)

4^n = 4^-1

n = -1

In each of the cases, we have applied the laws of the exponents as we know them.

Learn more about exponent:https://brainly.com/question/5497425

#SPJ1

The equation of the forms are matched as;

2⁻⁷/2ⁿ = 2², n = -9

6⁵ * 6ⁿ = 6. n = -4

(8ⁿ)⁷ = 8²¹, n = 3

4ⁿ = 1/4 , n = -1

What are index forms?

Index forms are simply described as mathematical forms that are used to represent numbers of variables that are too large or small.

To multiply index forms, you need to add the exponents of the same bases.

To divide index forms, you need to subtract the exponents of the same bases.

From the information given, we have that;

2⁻⁷/2ⁿ = 2²

cross multiply the values

2⁻⁷ = 2²⁺ⁿ

Then,

-7 = 2 + n

n = -9

6⁵ * 6ⁿ = 6

take the exponents

5 + n= 1

n =- 4

(8ⁿ)⁷ = 8²¹

We have;

7n = 21

Make 'n' the subject

n = 3

4ⁿ = 1/4

4ⁿ = 4⁻¹

n =-1

Learn about index forms at: https://brainly.com/question/15361818

#SPJ1

Penny needs 12 ounces of a snack mix that is made up of chocolate and almonds. Chocolate cost $3. 50 per ounce and almonds cost $4. 50 per ounce. Penny has $50 to spend and plans to sell it all. X the amount of chocolate and Y is the amount of almonds. Determine which equations you are used to form a system of equations for the scenario

Answers

The two equations which can be used to form a system of equations for the scenario are X + Y = 12 and 3.50X + 4.50Y = 50

To solve this problem, we need to form a system of equations. Let X be the amount of chocolate and Y be the amount of almonds. The first equation we can form is based on the total amount of snack mix that Penny needs, which is 12 ounces:

X + Y = 12

The second equation we can form is based on the cost of the ingredients. We know that chocolate costs $3.50 per ounce and almonds cost $4.50 per ounce. If X is the amount of chocolate and Y is the amount of almonds, then the total cost of the snack mix will be:

3.50X + 4.50Y = 50

This equation represents the fact that Penny has $50 to spend on the snack mix. Now we have a system of two equations that we can use to solve for X and Y. We can use substitution or elimination to solve the system and find the values of X and Y that satisfy both equations.

Once we have those values, we can check that they add up to 12 and that the total cost is $50. This system of equations allows us to calculate the amount of chocolate and almonds Penny needs to make the snack mix within her budget.

To know more about equations, visit:

https://brainly.com/question/29657983#

#SPJ11

The sum of five consecutive odd integers is 235. What is the greatest of
these integers?

Answers

Answer:

x + x + 2 + x + 4 + x + 6 + x + 8 = 235

5x + 20 = 235

5x = 215, so x = 43

The integers are 43, 45, 47, 49, and 51.

The greatest of these integers is 51.

Find the mean absolute deviation (MAD) of the data in the pictograph below. Baskets
The key says one basketball picture equals two baskets. The key says one basketball picture equals two baskets. A picture graph labeled Baskets each student made. The vertical axis is labeled Baskets made. The horizontal axis is labeled Student. The names from left to right on the horizontal axis are Reynaldo, Marcelle, Allie, and Fernando. There are two basketball pictures above Reynaldo. There are four basketball pictures above Marcelle. There are three basketball pictures above Allie. There are five basketball pictures above Fernando

Answers

The mean absolute deviation (MAD) of the data in the pictograph is equals to the one basketball.

We have a data in the pictograph. In mathematics, a pictograph is a pictorial representation of data using images, icons. It is also known as a pictogram. We have a pictograph, in which the vertical axis is labeled Baskets made and the horizontal axis is labeled Student. Here, one basketball picture equals two baskets. Mean absolute deviation (MAD) is a statistical measure of the average absolute distance between each data value and the mean of a data set. It is a parameter or statistic that measures the spread, or variation, in data.

Mean is defined as the sum of data values divided by number of values.

Sum of data values = 4 + 4×2 + 3×2 + 5×2

= 28

So, mean = 28/4 = 7

Now, | 4 - 7| + |8 - 7| + |6 -7 | + | 10 - 7|

= 3 + 1 + 1 + 3 = 8

So,, mean absolute deviation (MAD) of the data = 8/7 = 1.1 ~ 1. Hence, required value is 1.

For more information about mean absolute deviations, visit :

https://brainly.com/question/26462759

#SPJ4

Complete question :

Find the mean absolute deviation (MAD) of the data in the pictograph below. Baskets

The key says one basketball picture equals two baskets. The key says one basketball picture equals two baskets. A picture graph labeled Baskets each student made. The vertical axis is labeled Baskets made. The horizontal axis is labeled Student. The names from left to right on the horizontal axis are Reynaldo, Marcelle, Allie, and Fernando. There are two basketball pictures above Reynaldo. There are four basketball pictures above Marcelle. There are three basketball pictures above Allie. There are five basketball pictures above Fernando

in KLMO, OM-25. What are the coordinates of M and K?
L(2b+4c4d)

M

Answers

The value of coordinate M and K are,

M = (25, 0)

K = (2b + 4c - 25, 2d)

Given that;

In KLMO,

OM = 25

L = (2b + 4c , 4d)

Hence, We can formulate;

The value of coordinate of M is,

M = (25, 0)

Since, M lies on x - axis.

Let the coordinate of K is,

K = (x, y)

Hence, Midpoint of LO is same as midpoint of KM.

Midpoint of LO is,

(0 + 2b + 4c / 2, 4d/2)

(b + 2c, 2d)

Midpoint of KM is

(x + 25/2, y + 0/2)

(x + 25/2 , y/2)

By comparing,

x + 25/2 = b + 2c

x + 25 = 2b + 4c

x = 2b + 4c - 25

y/2 = 2d

y = 2d

Thus, the coordinate of K is,

K = (2b + 4c - 25, 2d)

Learn more about the coordinate visit:

https://brainly.com/question/24394007

#SPJ1

Suppose you are doing a 5000 piece puzzle. you have already placed 175 pieces. every minute you place 10 more pieces. after 50 minutes, how many pieces will you have placed

Answers

After 50 minutes, you will have placed 675 pieces

Given, you are doing a 5000 piece puzzle. You have already placed 175 pieces, and every minute you place 10 more pieces. After 50 minutes, we need to find how many pieces you will have placed.

We can start by finding the number of pieces you place in 50 minutes.

Every minute, you place 10 more pieces, so in 50 minutes, you will place:

10 x 50 = 500 pieces

Adding the pieces you have already placed, we get:

175 + 500 = 675 pieces

Therefore, after 50 minutes, you will have placed 675 pieces

Learn more about Puzzle here

https://brainly.com/question/27865156

#SPJ4

In Triangle JKL, ∠J is congruent to ∠L.

Answers

The measure of ∠L in Triangle JKL is 56.1 degrees.

What is the measures in triangles?

In geometry, the measures in triangles refer to the angles and sides within a triangle. Triangles are three-sided polygons, and the measures of their angles and sides are important properties that determine their shape and characteristics.

In a triangle, the sum of the measures of all three angles is always 180 degrees. Therefore, to find the measure of ∠L in Triangle JKL, we can use the information given:

∠J is congruent to ∠L, which means they have the same measure.

∠K is given as 67.8 degrees.

Since ∠J is congruent to ∠L, we can denote their measure as "x".

So, the sum of the measures of ∠J, ∠K, and ∠L is 180 degrees:

∠J + ∠K + ∠L = 180

Substituting the given values:

x + 67.8 + x = 180

Simplifying the equation:

2x + 67.8 = 180

Subtracting 67.8 from both sides:

2x = 180 - 67.8

2x = 112.2

Dividing both sides by 2:

x = 112.2 / 2

x = 56.1

Hence, the measure of ∠L in Triangle JKL is 56.1 degrees.

To learn more about measures in triangles, Visit

brainly.com/question/25215131

#SPJ1

Edwin fills 15 test tubes with a solution. each test tube contains 150 milliliters of solution.

how many liters of solution in all is there in the test tubes?


2.25 l

22.5 l

225 l

2,250 l

Answers

2.25 liters of solution in all is there in the test tubes. So, the correct option is 2.25 l.

The total volume of solution in the 15 test tubes can be calculated by multiplying the volume of one test tube by the number of test tubes:

Total volume = 15 test tubes × 150 milliliters/test tube

Total volume = 2,250 milliliters

However, the question asks for the answer in liters, so we need to convert milliliters to liters by dividing by 1,000:

Total volume = 2,250 milliliters ÷ 1,000

Total volume = 2.25 liters

Therefore, there are 2.25 liters of solution in all the test tubes.

Know more about solution here:

https://brainly.com/question/25326161

#SPJ11

If using the method of completing the square to solve the quadratic equation x^2-19x-39=0x 2 −19x−39=0, which number would have to be added to "complete the square"?​

Answers

We know that the solutions to the quadratic equation are x=21 or x=-12.

To solve the quadratic equation x^2-19x-39=0 using the method of completing the square, the number that would have to be added to "complete the square" is 91.

First, move the constant term to the right side: x^2-19x=39.

Then, take half of the coefficient of x, square it, and add it to both sides: x^2-19x+90.25=129.25.

This can be factored as (x-9.5)^2=129.25.

Taking the square root of both sides, we get x-9.5=±√129.25.

Solving for x, we get x=9.5±√129.25, which simplifies to x=9.5±11.5.

Therefore, the solutions to the quadratic equation are x=21 or x=-12.

To know more about quadratic equation refer here

https://brainly.com/question/17177510#

SPJ11

In def x is a point on ef and y is a point on df so that xy ||de if xf =10 yf=5 and ef = 13 what is dy

Answers

DF^2 = 170 * 169^2 / (135^2 + 200 - 26EX) = 106.7027

DY^2 = 170 * 169^2 / (135^2 + 50 -

In the given figure, we have a triangle DEF, where EF is a transversal intersecting DE and DF at points X and Y, respectively, such that XY || DE.

    D

   / \

  /   \

 /     \

E-------F

Given that XF = 10, YF = 5, and EF = 13, we need to find DY.

We can start by using the property of similar triangles. Since XY || DE, we have the following similarity ratios:

EF / ED = EY / EJ  (where J is the intersection of XY and DF)

EF / DF = EJ / EY

Substituting the given values, we get:

13 / ED = EY / EJ

13 / DF = EJ / (13 - EY)

Multiplying the above two equations, we get:

13 / ED * 13 / DF = EY / EJ * EJ / (13 - EY)

169 / (ED * DF) = EY / (13 - EY)

Substituting the values of XF = 10 and YF = 5, we get:

169 / (ED * DF) = 5 / 8

ED / DF = 135 / 169

Using the Pythagorean theorem on triangles DEX and DFY, we get:

ED^2 = EX^2 + DX^2

DF^2 = FY^2 + DY^2

Since EX + DX = EF = 13, we have DX = 13 - EX. Substituting this in the first equation and simplifying, we get:

ED^2 = EX^2 + (13 - EX)^2

ED^2 = 2EX^2 - 26EX + 170

Similarly, substituting FY = 13 - EY in the second equation and simplifying, we get:

DF^2 = FY^2 + DY^2

DF^2 = 170 - 26EY + EY^2 + DY^2

Now, using the fact that ED/DF = 135/169, we can substitute ED^2 = (135/169)^2 * DF^2 in the above equation for ED^2, and simplify to get:

(135/169)^2 * DF^2 = 2EX^2 - 26EX + 170

DF^2 = 170 * 169^2 / (135^2 + 2EX^2 - 26EX)

DF^2 = 170 * 169^2 / (135^2 + 2(10^2) - 26EX)   (Substituting XF = 10)

Similarly, we can substitute EY = 5 in the above equation for DF^2 and simplify to get:

FY^2 + DY^2 = 170 * 169^2 / (135^2 + 2(5^2) - 26EY)   (Substituting YF = 5)

DY^2 = 170 * 169^2 / (135^2 + 2(5^2) - 26EY) - FY^2

Substituting the given values, we get:

DF^2 = 170 * 169^2 / (135^2 + 200 - 26EX) = 106.7027

DY^2 = 170 * 169^2 / (135^2 + 50 -

To learn more about equations visit:

https://brainly.com/question/11950137

#SPJ11

Why do you think that credit cards tend to be the entry point for establishing credit for so many consumers?

Answers

I believe credit cards tend to be the entry point for establishing credit for so many consumers because they provide an easy and accessible way for individuals to begin building their credit history. Credit card companies report to credit bureaus on a regular basis, which helps establish a credit score and credit history.

Additionally, credit cards offer a convenient way for individuals to make purchases and build their credit at the same time. However, it is important for individuals to use their credit cards responsibly and make timely payments in order to maintain good credit standing.

To know more about think refer here

https://brainly.com/question/3368793#

#SPJ11

What is the x intercept of f(x)= 2x^2+5x+3

Answers

Answer: x intercepts = (-1.5,0) and (-1,0)

Step-by-step explanation: Graphed it in desmos :)

Find the next term in each sequence.
Question 1:
35, 29, 23, 17, ?.
Question 2:
1, 2, 5, 10, ?.
Please Include an Explanation of how to solve problems like this!
Thanks a ton!

Answers

1. For the sequence : 35, 29, 23, 17, ?; the next term is 11

2. For the sequence : 1, 2, 5, 10, ?; the next term is 17

Calculating the term in a sequence

From the question, we are to calculate the next term in each of the given sequence

From the given sequence,

35, 29, 23, 17, ?.

To determine the next term, we will determine the common difference

Common difference = Second term - First term

Common difference = 29 - 35

Common difference = -6

Thus,

To determine the next term, we will add the common difference to the last term

That is,

17 + - 6 = 17 - 6

= 11

The next term is 11

For the sequence 1, 2, 5, 10, ?.

Common difference = successive odd numbers

To get the second term, we will add to the first term the first natural odd number

To get the third term, we will add to the second term the second natural odd number

And so on.

In the given sequence, we are to determine the 5th term

Thus,

We will add to the fourth term, the fourth natural odd number

That is,

10 + 7 = 17

Hence, the next term is 17

Learn more on Calculating the term in a sequence here: https://brainly.com/question/28583635

#SPJ1

Una persona observa una torre desde una distancia de 100m con un angulo de elevación de 70, con que función trigonométrica obtendrías la altura de la torre? Calcula la altura de la torre

Answers

The height of the tower is: 274.7m

How to solve

To find the height of the tower, we will use the tangent trigonometric function.

The tangent function relates the angle of elevation to the ratio of the opposite side (height of the tower) to the adjacent side (distance from the observer to the tower).

In this case, the angle of elevation is 70°, and the distance from the observer to the tower is 100 meters.

The formula we will use is:

tan(θ) = opposite / adjacent

tan(70°) = height / 100m

To calculate the height, we will rearrange the formula:

height = 100m * tan(70°)

Using a calculator, we find that tan(70°) ≈ 2.747.

Therefore, the height of the tower is: 274.7m

height ≈ 100m * 2.747 ≈ 274.7m

Read more about height here:

https://brainly.com/question/1739912

#SPJ1

The question in English is:

A person observes a tower from a distance of 100m with an elevation angle of 70, with which trigonometric function would you obtain the height of the tower? Calculate the height of the tower

Find the work done by the force field F(x,y) = x^2i – ryj in moving a particle along the F semicircle y = Sqrt(4 – x^2) from P(2,0) to Q(-2,0) and then back along the line segment from Q to P.

Answers

The work done by the force field F along the semicircle and the line segment is 32/3.

The work done by a force field F along a curve C from point A to point B is given by the line integral:

W = ∫ F dot dr

where dot represents the dot product and dr is the differential displacement vector along the curve C.

Let's divide the curve C into two parts: the semicircle from P to Q, denoted by C1, and the line segment from Q to P, denoted by C2.

For C1, the curve can be parameterized as x = 2cos(t) and y = 2sin(t) for t in [0, pi]. The differential displacement vector is then given by:

dr = (-2sin(t) dt)i + (2cos(t) dt)j

The force field F(x,y) = x^2i - ryj, so we have:

F(x,y) = (2cos^2(t))i - (2rsin(t))j

The dot product F dot dr is then:

F dot dr = (2cos^2(t))(-2sin(t) dt) + (2rsin(t))(2cos(t) dt)

= -4cos^2(t)sin(t) dt + 4rcos(t)sin(t) dt

= 4sin(t)cos(t)(r - cos(t)) dt

Therefore, the work done along C1 is:

W1 = ∫ C1 F dot dr

= ∫[0, pi] 4sin(t)cos(t)(r - cos(t)) dt

This integral can be evaluated using the substitution u = cos(t), du = -sin(t) dt:

W1 = -∫[1, -1] 4u(r - u) du

= 4r∫[1, -1] u du - 4∫[1, -1] u^2 du

= 0

Hence, the work done along C1 is 0.

For C2, the curve is simply the line segment from Q(-2,0) to P(2,0), which is parallel to the x-axis. Therefore, the differential displacement vector is given by:

dr = dx i

where i is the unit vector in the x-direction. The force field is the same as before, F(x,y) = x^2i - ryj. Along C2, y = 0, so the force field reduces to:

F(x,y) = x^2i

The dot product F dot dr is then:

F dot dr = x^2 dx

Therefore, the work done along C2 is:

W2 = ∫ C2 F dot dr

= ∫[-2, 2] x^2 dx

= 32/3

Hence, the work done along C2 is 32/3.

The total work done along the curve C is the sum of the work done along C1 and C2:

W = W1 + W2 = 0 + 32/3 = 32/3

Therefore, the work done by the force field F along the semicircle and the line segment is 32/3.

To learn more about dot product visit: https://brainly.com/question/29097076

#SPJ11

if you increase your reading speed so that each page takes you 30 seconds less than it did before, and you begin reading 20 minutes per day, how many 200 page books can you now read in a year

Answers

We can read 91 books in a year if you increase your reading speed so that each page takes you 30 seconds less than it did before.

How is the number of books calculated?

Now If each page now takes 30 seconds less to read than before,

then you will save 30*200 = 6000 seconds (or 100 minutes) on each book.

So, the time it will take you to read a 200-page book will be

20 minutes - 100 minutes = -80 minutes,

which means you will finish a 200-page book in 80 minutes (or 1 hour and 20 minutes).

In a year, there are 365 days. If you read for 20 minutes per day, then you will read for a total of

365 * 20 = 7300 minutes (or 121.67 hours) in a year.

Since you can finish a 200-page book in 80 minutes, you can read 7300/80 = 91.25 books in a year.

However, since you cannot read a fraction of a book, the maximum number of 200-page books you can read in a year is 91 books.

To know more about fraction visit:

brainly.com/question/10354322

#SPJ1

Complete question is :

If you increase your reading speed so that each page takes you 30 seconds less than it did before,

and you begin reading 20 minutes per day,

How many 200 page books can you now read in a year?

Suppose $40,000 is deposited into an account paying 2. 5% interest, compounded continuously.



How much money is in the account after eight years if no withdrawals or additional deposits are made?

Answers

The formula for calculating the amount of money in an account with continuous compounding is:

[tex]A = Pe^{(rt)}[/tex]

where A is the amount of money in the account, P is the principal (initial deposit), e is the mathematical constant e (approximately equal to 2.71828), r is the interest rate (expressed as a decimal), and t is the time (in years).

Plugging in the given values, we get:

A =[tex]40000 * e^{(0.025 * 8)[/tex]

Using a calculator, we find that [tex]e^{(0.025 * 8)[/tex] is approximately 1.2214, so:

A = 40000 * 1.2214 = $48,856.12

Therefore, the amount of money in the account after eight years with continuous compounding is $48,856.12.

To know more about interest rate refer here

https://brainly.com/question/14445709#

#SPJ11

Other Questions
Which of the following are areas of sectors formed by Angle ABC?B = 86.4AB=4.1cm Select all of the sampling techniques that lead to an unbiased sample.cluster samplingover-samplingstratified random samplingsystematic samplingmultistage sampling Calculating the Variable Overhead Spending and Efficiency Variances Standish Company manufactures consumer products and provided the following information for the month of February: Units produced 131,000 Standard direct labor hours per unit 0. 20 Standard variable overhead rate (per direct labor hour) $3. 40 Actual variable overhead costs $88,670 Actual hours worked 26,350Required:1. Calculate the total variable overhead variance. 2. What if actual production had been 129,500 units? Imagine you are paul and write a letter to a friend in houston in which you describe your new community and some of the people you have met. also, be sure to include your thoughts and feelings about lake windsor downs. tangerine novel A circular pool has a radius of 6 ft. What is the approximate circumference of the pool?A. 19 ftB. 38 ftC. 113 ftD. 452 ft A person completes 68 km in 50 minutes via Jeep. Starting 20 minutes, he travels by x km/hr and next 25 minutes by 2x km/hr and rest time by 3x km/hr. What is the value of x ? What term is used to describe the period where philosophers like John Locke andMontesquieu described their thoughts on the nature of humanity and government? The strength of a beam is proportional to the width and the square of the depth. A beam is cut from a cylindrical log of diameter d = 24 cm. The figures show different ways this can be done. Express the strength of the beam as a function of the angle in the figures. (Use k as your proportionality constant.) Critical ThinkingNick is an avid golfer, but lately he has been experiencing severe pain in his elbow. Discuss an injury reviewed in this chapter that Nick may be suffering from. What type of treatment would you suggest? I NEED HELP ASAP with a science experiment give me ideas what experiments (mini-labs) I could do based on the task! READ THE SCREENSHOT Ignatio and his two friends each bought a ticket to play lazer tag and each spent $15 on game tokens at Fun Mountain. Let t represent the cost of a lazer tag ticket. Enter an expression that represents the total amount that Ignatio and his friends spent. What is the process of carbon dioxide getting into the atmosphere Can someone help me to answer these questions please I would appreciate it.Chapter 13/14. The Civil War. Watch the Hari Jones TED talk. Then, answer the following questions in 7-10 sentences.In serving in the military, what contributions did African Americans make to the Union war effort?What kinds of discrimination did they face? What kind of heroism did they exhibit, and when?Write a reaction to Dr/ Jones' TED talk. What did you know/not know? What surprised/amazed/angered/made you proud/saddened/made you happy? What are the slopes and y-intercept of a graph? Population ecologists follow the fate of same-age cohorts to. If BA = 5x + 5 and AD = 10x - 20, find BD. It is a parallelogram by the way. a laundromat has 5 washing machines. a typical machine breaks down once every 5 days. a repairer can repair a machine in an average of 2.5 days. currently, three repairers are on duty. the owner of the laundromat has the option of replacing them with a superworker, who can repair a machine in an average of 5 6 day. the salary of the superworker equals the pay of the three regular employees. breakdown and service times are exponential. should the laundromat replace the three repairers with the superworker? Your reaction as a tourism professional if you have a colleague or customer who either accepts or does not accept it Circle a has a radius of 4.0 cm. the shortest distance between b and c on the circle is 3.5 cm. what is the approximate area of the shaded portion of circle a? 7.0 cm 14.4 cm 25.15 cm 50.3 cm What was the purpose of the Great Purges under Stalin?to eliminate any and all opposition to Stalin's controlto eliminate any and all Christians in Russiato eliminate Nazi control of Germanyto eliminate any and all capitalists in Russiato eliminate Jews in Russia?